0% found this document useful (0 votes)
6 views12 pages

Eisenbud-Chapter 1

The document contains exercises and solutions related to modules over commutative rings, specifically focusing on Noetherian modules and their properties. It includes proofs of the equivalence of various conditions for Noetherian modules, as well as discussions on minimal primes in Noetherian rings and irreducibility of algebraic sets. Additionally, it explores the relationship between algebraic sets and polynomial rings, providing examples and proofs of bijections between different algebraic structures.

Uploaded by

zulysalinas
Copyright
© © All Rights Reserved
We take content rights seriously. If you suspect this is your content, claim it here.
Available Formats
Download as PDF, TXT or read online on Scribd
0% found this document useful (0 votes)
6 views12 pages

Eisenbud-Chapter 1

The document contains exercises and solutions related to modules over commutative rings, specifically focusing on Noetherian modules and their properties. It includes proofs of the equivalence of various conditions for Noetherian modules, as well as discussions on minimal primes in Noetherian rings and irreducibility of algebraic sets. Additionally, it explores the relationship between algebraic sets and polynomial rings, providing examples and proofs of bijections between different algebraic structures.

Uploaded by

zulysalinas
Copyright
© © All Rights Reserved
We take content rights seriously. If you suspect this is your content, claim it here.
Available Formats
Download as PDF, TXT or read online on Scribd
You are on page 1/ 12

Modern Algebra I

November 19, 2024

Homework 01
Unlike in the textbook, we use ⊆ to denote the containment of two sets (for the simple fact that it is a
habit to use this symbol).
Exercise 1.1. Prove that the following conditions on a module M over a commutative ring R
are equivalent (the fourth is Hilbert’s original formulation; the first and third are the ones most often
used). The case M = R is the case of ideals.

1. M is Noetherian (that is, every submodule of M is finitely generated).

2. Every ascending chain of submodules of M terminates (“ascending chain condition”).

3. Every set of submodules of M contains elements maximal under inclusion.

4. Given any sequence of elements f1 , f2 , . . . ∈ M, there is a number m such that for each n > m
there is an expression fn = ∑m
i=1 ai f i with ai ∈ R.

Solution. (1. ⇒ 2.) Suppose that M is Noetherian. Let J1 ⊆ J2 ⊆ J3 ⊆ · · · be an ascending chain of


submodules of M. We know that [
J := Jk
k∈N

is also a submodule of M. By hypothesis, J is finitely generated, so there exist f1 , . . . , fn ∈ J such that

J = ( f1 , . . . , fn ).

Since (Jk )k∈N is an ascending chain, there exists n0 ∈ N such that

fk ∈ Jn0 for all k = 1, . . . , n.

Thus, the ascending chain of submodules terminates at n0 . Since (Jk )k∈N is arbitrary, we have proved
that every ascending chain of submodules of M terminates.
(2. ⇒ 3.) Suppose that every ascending chain of submodules of M terminates. Let Θ = (Jα )α be
a set of submodules of M. By contradiction, suppose that Θ does not have a maximal element. Let
Jα1 ∈ Θ be arbitrary. By our assumption, Jα1 is not maximal; so, there exists Jα2 ∈ Θ such that

Jα1 ⊆ Jα2 .

Now, since Jα2 is not maximal, there exists J3 ∈ Θ such that

Jα2 ⊆ Jα3 .

By doing this, we can find an ascending chain (Iαk )k∈N of submodules of M. By hypothesis, (Iαk )k∈N
terminates, thus there exists k0 ∈ N such that

Iαk ⊆ Iαk0 for all k ∈ N.

This means that Iαk0 is a maximal element in Θ, which is a contradiction. Since Θ is arbitrary, we
have shown that every set of submodules of M contains elements maximal under inclusion.
(3. ⇒ 4.) Assume that every set of submodules of M contains elements maximal under inclusion.
Let ( fk )k∈N be a sequence of points in M. We consider the set of ideals

Θ := {( f1 , . . . , fk ) : k ∈ N},

which admits a maximal element. Let ( f1 , . . . , fk0 ) denote the maximal element of Θ. Since

( f1 , . . . , fk ) ⊆ ( f1 , . . . , fk+1 ) for all k ∈ N,

we see that
( f1 , . . . , fk0 ) = ( f1 , . . . , fk ) for all k ∈ N, k ≥ k0 .
Precesily, this means that, for each k ≥ k0 , there are a1 , . . . , ak0 ∈ R such that
m
f k = ∑ ai f i .
i=1

(4. ⇒ 1.) Suppose that any sequence of elements f1 , f2 , . . . ∈ M, there is a number m such that
for each n > m there is an expression fn = ∑m i=1 ai f i with ai ∈ R. Let J ⊆ M be a submodule. By
contradiction, suppose that J is not finitely generated. Let f1 ∈ J be fixed. Since J is not finitely
generated, there exists f2 ∈ J such that
f2 ∈
/ ( f1 ).
By the same argument, we can find f3 ∈ J such that

f3 ∈
/ ( f1 , f2 ).

Repeating this reasoning over and over again, we get a sequence ( fk )k∈N of points in J such that

fk+1 ∈
/ ( f1 , . . . , fk ) for all k ∈ N.

This means that the sequence ( fk )k∈N does not satisfy statement 4. of this result, which is a contra-
diction. So, we conclude that J is finitely generated. Since J is arbitrary, we have proved that M is
Noetherian.

Exercise 1.2. (Emmy Noether): Prove that if R is Noetherian, and I ⊆ R is an ideal, then among
the primes of R containing I there are only finitely many that are minimal with respect to inclusion
(these are usually called the minimal primes of I, or the primes minimal over I) as follows: Assuming
that the proposition fails, the Noetherian hypothesis guarantees the existence of an ideal I maximal
among ideals in R for which it fails. Show that I cannot be prime so that we can find elements f and
g in R, not in I, such that f g ∈ I. Now show that every prime minimal over I is minimal over one of
the larger ideals (I, f ) and (I, g).
With Hilbert’s basis theorem and the Nullstellensatz (see Exercise 1.9), Exercise 1.2 gives one of
the fundamental finiteness theorems of algebraic geometry: An algebraic set can have only finitely
many irreducible components. Originally the result was proved by difficult inductive arguments and
elimination theory. For a further discussion of the significance of this result see the beginning of
Chapter 3, and particularly example 2 there. The result of this exercise is strengthened in Theorem
3.1.

Solution. We proceed by contradiction, assuming that the set of ideals of R for which the statement
fails is not empty; we denote this set by Θ. Since Θ is a set of ideals in the Noetherian ring R, Θ
admits a maximal element I.
First, we prove that I is not prime. By contradiction, if I is prime, then I is an element in Θ with
a unique prime containing I, which is minimal with respect to inclusion (of course, we are talking
about I itself!). This contradicts our hypothesis, so I is not prime.
Now, since I is not prime, there exist f , g ∈ R\I such that f g ∈ I. For any of the infinite prime
ideals J containing I, f g ∈ J. Therefore, either

( f , I) ⊆ J,

or
(g, I) ⊆ J.
Without loss of generality, we assume that ( f , I) ⊆ J. Since there are infinitely many of Js, ( f , I) ⊆ J
implies that ( f , I) ∈ Θ. In addition, since

I ⊆ ( f , I),

we conclude that ( f , I) is an element in Θ, bigger than the maximal! Since we have reached a contra-
diction, the exercise statement is true.

Exercise 1.9. Let S = k[x1 , . . . , xr ], with k an algebraically closed field. Show that under the
correspondence of radical ideals in S and algebraic subsets of Ar , the prime ideals correspond to the
algebraic sets that cannot be written as a proper union of smaller algebraic sets (irreducible).

Solution. Suppose that J is a prime ideal. We need to prove that Z(J) is irreducible. By contradiction,
we assume that Z(J) is not irreducible, i.e., there exist J1 , J2 ◁ S such that

Z(J) = Z(J1 ) ∪ Z(J2 ). (1)

Since J is radical, by the Nullstellensatz we see that

J = rad (J)
= I(Z(J))
= I(Z(J1 ) ∪ Z(J2 ))
= I(Z(J1 )) ∩ I(Z(J2 )). (2)

But we know that J is prime, so by (2) we must have that

J = I(Z(J1 )) or J = I(Z(J2 )).

This, together with the proper union given in (1), implies that either

Z(J2 ) ⊆ Z(J1 ),

or
Z(J1 ) ⊆ Z(J2 ),
which is not possible! So, we conclude that Z(J) is irreducible.

Exercise 1.10. Find rings to represent the following figures:

The first represents the union of a circle and a parabola in the plane, and the second shows the union
of two skew lines in 3-space. (You may use the Nullstellensatz to prove your answer is right.)

Proof. For the first figure, we shall assume that the coordinate axes are displayed so that the circle
has radius 1 and, its center is in the point (0, 1).
Thus, this circle is associated with the equation

x2 + (y − 1)2 = 1.

For the parabola, we consider the points satisfying the equation


1
y = x2 .
4
The election of 1/4 guarantees that the circle and the parabola intersect in only one point, (0, 0).
Hence, we obtain the ring
k(x, y)/(x2 + (y − 1)2 − 1) ∩ (x2 − 4y).
For the second figure, we shall focus on spatial lines. We know that any line L in the 3-dimensional
space is parameterized by

R → R3
t 7→ (a, b, c)t + (a0 , b0 , c0 ),

where at least one of the entries in (a, b, c) is nonzero. Without loss of generality, we assume that
a ̸= 0. Any point (x, y, z) in L is such that

x = at + a0 , y = bt + b0 , z = ct + c0 ,

for some t. From this, we see that


x − a0
t= ,
a
and then, 
 y = x − a0
b + b0 ,
a
 z = x − a0 c + c .
0
a
After a few simple calculations, we see that every point (x,y,z) in L satisfies the equations
 L
R (x, y) := b(x − a0 ) − a(y − b0 ) = 0,
SL (x, z) := c(x − a0 ) − a(z − c0 ) = 0.

Now, for our skew lines, which we call L1 and L2 , we associate the corresponding RL1 , SL1 , RL2 , SL2
(depending on two variables, let us say x and y). So, the second figure corresponds to the ring

k[x, y]/(RL1 (x, y), SL1 (x, y), RL2 (x, y), SL2 (x, y)).

Exercise 1.10. When we draw pictures representing algebraic sets, we often draw the same
picture for all ground fields k, although by rights it generally represents best the case k = R. In fact,
we are usually interested in the case of an algebraically closed field k, such as k = C, where the
Nullstellensatz applies. The main way in which the pictures can be misleading is illustrated by the
following examples.
a. If k = R then the ring k[x, y]/(x2 + y2 − 1) ∩ (y − 2 − x2 ) corresponds to the union of a circle and a
parabola.
If k = C, show that there are four points in the intersection of these two components. Show that
there is a bijection given by polynomial maps between the parabola and the line x = 0. Show
further that “projection from the north pole” gives a bijection (given by rational functions)
between the circle minus one point and the line minus two points. (Can you find such a bijection
between the circle and the line minus one point?)

b. Show that the polynomial f (x, y) = y2 − (x − 1)x(x + 1) is irreducible over any field k. Thus
X = Z( f ) ⊆ A2 is an irreducible algebraic set. This is not so obvious from the real picture,
which approximately resembles the following image.

Solution of a. To find the points of the intersection, we must find the solutions to the system
 2
x + y2 − 1 = 0,
y − 2 − x2 = 0.

When replacing y = x2 + 2 in the first equation of our system, we obtain

x2 + (x2 + 2)2 = 1.

We write z := x2 , so our problem is reduced to solve the equation

z + (z + 2)2 = 1,

or equivalently, we have to find the roots of

z2 + 5z + 3 = 0.

It is not difficult to see that such roots are


√ √
−5 + 13 −5 − 13
z1 := and z2 := .
2 2
With these numbers, we find the solutions of z = x2 :
√ √ √ √
x1 := z1 , x2 := − z1 , x3 := z2 , x4 := − z2 ,

which gives us also the solutions of y = x2 + 2:

y1 := x12 + 2, y2 := x22 + 2, y3 := x32 + 2, y4 := x42 + 2.

These calculations show that there are four points in the intersection of the two components in our
figure.
Now, for the bijection between the parabola and the line x = 0, we consider the map

f :R⊆C∼
= R2 → C2 ∼
= R2
(0,t) 7→ f (t) = (t 2 ,t).

If f (t1 ) = f (t2 ), then t1 = t2 ; so, f is injective. In addition, we see that

f (R) = {(t,t 2 ) : t ∈ R}.

This means that f is a bijection between the parabola {(t,t 2 ) : t ∈ R} and the line x = 0.
For the last part, let us consider the stereographic projection1
x
(x, y) 7→ ,
1−y
with inverse
2u u2 − 1
 
u 7→ , .
u2 + 1 u2 + 1
In our context, the above map is defined over all the complex numbers, except for i and −i. Both
values remain outside the domain of the stereographic projection. Following the suggestions given
by the textbook, we shall move i to infinity by using the change of variables:
1
z := + i.
u
Since
1
u= ,
z−i
for any complex z ̸= i, we obtain

2u 2(z − i)
= ,
u2 + 1 1 + (z − i)2
u2 − 1 1 − (z − i)2
= .
u2 + 1 1 + (z − i)2

This shows that


2(z − i) 1 − (z − i)2
 
z 7→ ,
1 + (z − i)2 1 + (z − i)2
is a bijection between the circle and the line minus one point.

Solution of b. By contradiction, suppose that there exists polynomials g, h ∈ k[x, y] such that

f (x, y) = y2 − (x − 1)x(x + 1) = g(x, y)h(x, y). (3)

We consider two cases:


degy g = 1 and degy g = 2.
If degy g = 1, then degy h = 1. Thus, there exist p, q, r, s ∈ k[x] such that

g(x, y) = p(x)y + q(x),


h(x, y) = r(x)y + s(x).
1 For reference, see Elementary Differential Geometry–Example 6.3.5, by Andrew Pressley.
By using (3), we deduce that

p(x)r(x) = 1,
s(x) + q(x) = 0,
q(x)s(x) = −(x − 1)x(x + 1).

Since s(x) = −q(x), we see that

q(x)2 = −q(x)s(x) = (x − 1)x(x + 1)

is a polynomial of even degree, which is not true. So, it is not possible that degy g = 1.
Now, if degy g = 2, then degy h = 0. In consequence, there exist p, q, r, s ∈ k[x] such that

g(x, y) = p(x)y2 + q(x)y + r(x),


h(x, y) = s(x).

In this case, by (3), it follows that

p(x)s(x) = 1,
q(x)s(x) = 0,
r(x)s(x) = −(x − 1)x(x + 1).

By the first equation we see that s(x) is a unit, so it must be a constant polynomial. In other words,
degs = 0. But h(x, y) = s(x), so degx h = degy h = 0, so h is constant, which can be assumed to be 1.
Thus, (3) is not factorized.
Summarizing, we have proved that f is irreducible over any field k. We know that irreducible
elements generate prime ideals; but prime ideals are in correspondence with irreducible algebraic
sets, as we showed in Exercise 1.9. Thus, X = Z( f ) ⊆ A2 is an irreducible algebraic set.

Exercise 1.13. Suppose that I is an ideal in a commutative ring. (Part 1) Show that rad (I)
is finitely generated, then for some integer N we have (rad (I))N ⊆ I. (Part 2) Conclude that, in a
Noetherian ring, the ideals I and J have the same radical if and only if there is some integer N such
that I N ⊆ J and J N ⊆ I. (Part 3) Use the Nullstellensatz to deduce that if I, J ⊆ S = k[x1 , . . . , xr ] are
ideals and k is algebraically closed, then Z(I) = Z(J) if and only if I N ⊆ J and J N ⊆ I for some N.

Solution. Part 1. Suppose that rad (I) is finitely generated; thus, there exist f1 , . . . , fn ∈ rad (I) such
that
rad (I) = ( f1 , . . . , fn ).
For each j = 1, . . . , n, we write
I j := ( f1 , . . . , f j ).
By induction over j, we want to prove that for each j there exists N j ∈ N such that
N
I j j ⊆ I. (4)

Induction basis: Let j equal one. Since f1 ∈ rad (I), there exists N1 ∈ N such that

f1N1 ∈ I.

From this fact, it is not difficult to see that

I1N1 = ( f1 )N1 ⊆ I.
This last line shows that (4) is true for j = 1.
Induction step: Let j > 1 be fixed. Suppose that (4) holds for j − 1. So, there exists N j−1 ∈ N
such that
N j−1
I j−1 ⊆ I.
Now, since f j ∈ rad (I), there is a number m ∈ N such that

f jm ∈ I.

Note that I j−1 ⊆ I j ; so, for each p ∈ I j there exist q p ∈ I j−1 and t ∈ R such that

p = qp + t f j.

By applying the binomial formula to the above identity, we see that


 
N j−1 +m N j−1 + m
pN j−1 +m = (q p + t f j ) =∑ qip (t f j )N j−1 +m−i .
i
i

Therefore, pN j−1 +m belongs to I because f jm ∈ I. By taking N j := N j−1 + m, we see that pN j ∈ I, and


in consequence,
p ∈ rad (I).
Since p and j are arbitrary, we have proved that
N
Ij j ⊆ I for all j = 1, . . . , n,

completing the induction.


In particular, it follows that

InN = ( f1 , . . . , fn )N = (rad (I))N ⊆ I,

for some N = (Nn ) ∈ N.


Part 2. Now, let R be a Noetherian ring. Let I, J be ideals in R. Since R is Noetherian, I and J are
finitely generated.
First, we assume that rad (I) = rad (J). By Part 1 in this proof, we know that there exist M1 , M2 ∈ N
such that
(rad (I))M1 ⊆ I and (rad (J))M2 ⊆ J.
By taking N := max{M1 , M2 }, we see that

(rad (I))N ⊆ I and (rad (J))N ⊆ J.

Now, by definition, we have that

I ⊆ rad (I) and J ⊆ rad (J),

and, as a consequence, it follows that

I N ⊆ (rad (I))N and J N ⊆ (rad (J))N .

When combining all these observations, we obtain

I N ⊆ (rad (I))N = (rad (J))N ⊆ J,


J N ⊆ (rad (J))N = (rad (I))N ⊆ I.

These contentions conclude the first part of the proof.


Now, we suppose that there exists N ∈ N such that

IN ⊆ J and J N ⊆ I. (5)

Let p ∈ rad (I) be arbitrary. There exists m ∈ N such that

pm ∈ I.

By (5), we see that


pmN ∈ I N ⊆ J,
which means that p ∈ rad (J). Since p is arbitrary, we conclude that

rad (I) ⊆ rad (J).

Similarly, we can verify that rad (J) ⊆ rad (I). Therefore,

rad (I) = rad (J).

Part 3. Finally, by the Nullstellensatz, if Z(I) = Z(J), then

rad (I) = I(Z(I)) = I(Z(J)) = rad (J).

By Part 2. in this proof, we know there exists N ∈ N such that

IN ⊆ J and J N ⊆ I.

Exercise 1.24. Spec, max-Spec, and the Zariski Topology.


An ideal I ⊆ R is called a prime ideal if R/I is an integral domain. I is a maximal ideal if R/I is a
field, so maximal ideals are prime. The set of all prime ideals of a ring R is called the spectrum of R,
written Spec R, and the set of all maximal ideals is usually denoted by the typographically awkward
but reasonably descriptive name max − Spec R.
In the text we defined the Zariski topology on an algebraic set over any algebraically closed field
k. We may identify X with the set max − Spec A(X). The subset Z(I) is identified with the set of
maximal ideals containing I. This suggests a way of defining a topology on the set of maximal ideals
of any ring. The corresponding idea can also be applied to the set of all prime ideals, and it turns
out to be even more useful there. These ideas were first pursued by Oscar Zariski, and the resulting
topology bears his name.

Definition. Let R be any ring. The subsets of Spec R of the form

Z(I) := {p a prime ideal of R ; p ⊇ I},

for ideals I of R are called Zariski-closed subsets. When there is no danger of confusion, we shall
simply call them closed subsets.

a. Prove that finite unions and arbitrary intersections of closed subsets are closed, and therefore the
closed subsets define a topology, called the Zariski topology, on Spec R. The induced topology
on the subset max-Spec R is also called the Zariski topology.
If k = R or C (or some other topological field), then we have two topologies on kn : the topology
induced from the topology of k, called the classical topology, and the Zariski topology. The
Zariski topology has many fewer closed sets than the classical topology.
b. Suppose for simplicity that k is an algebraically closed field, in the Zariski topology on Al (k) (that
is, on the maximal ideals of k[x]) show that the open sets are exactly the complements of finite
sets. In particular, this topology is not Hausdorff. Show that the Zariski topology on An (k) = kn
is not the product topology, even for n = 2.

c. We define a distinguished open set of Spec R to be an open set of the form

U( f ) := {p a prime ideal of R ; p ⊇ I}

for some f ∈ R. (Part 1) Show that the distinguished open sets form a basis for the Zariski
topology, in the sense that every open set is a union of distinguished open sets. (Part 2) Show
S
that Spec R = i U( fi ) for some collection fi of elements of R if and only if the ideal generated
by all the fi is the unit ideal (1).

d. Show that if R is any ring then Spec R is compact in the Zariski topology (that is, every open
covering has a finite refinement).

Solution of a. Note that Z(0) = Spec R and Z(1) = 0.


Unions: For simplicity, we work with two closed subsets Z(I) and Z(J), where I and J are ideals
in R. We shall prove that Z(I) ∪ Z(J) is also closed; indeed, we shall show that Z(I) ∪ Z(J) = Z(IJ).
Let p ∈ Z(I) ∪ Z(J) be arbitrary. Without loss of generality, we assume that p ∈ Z(I). Thus, p is
a prime ideal of R such that
I ⊆ p.
Since I is an ideal, IJ ⊆ I. Thus, IJ ⊆ p, which means that p ∈ Z(IJ). Since, p is arbitrary, we get

Z(I) ∪ Z(J) ⊆ Z(IJ). (6)

Now, let p ∈ Z(IJ) be arbitrary. This means that

IJ ⊆ p. (7)

By contradiction, suppose that I ⊈ p and J ⊈ p. Hence, we can take points a ∈ I\p and b ∈ J\p. Note
that ab ∈ IJ, so, by (7) we have that ab ∈ p, but precisely this is impossible! Thus, p ∈ Z(I) ∪ Z(J)
for every p ∈ Z(IJ). In conclusion, we have that

Z(IJ) ⊆ Z(I) ∪ Z(J). (8)

By combining (6) and (8), it holds that

Z(I) ∪ Z(J) = Z(IJ).

Intersections: If (Z(Iα ))α is a family of algebraic sets, it holds that


!
\ [
Z(Iα ) = Z Iα . (9)
α α

If p ∈
T
α Z(Iα ), then
Iα ⊆ p ∀α.
Hence, we obtain [
Iα ⊆ p,
α

which means that p ∈ Z ( Now, if p ∈ Z (


S S
α Iα ). α Iα ), then
[
Iα ⊆ Iα ⊆ p ∀α.
α
In other words, p ∈ Iα for every α, and in consequence,
\
p∈ Z(Iα ).
α

These observations prove (9).

Solution of b. We shall show that the closed sets are finite, i.e., algebraic sets are finite. Let J be an
ideal in k[x]. Note that k[x] is a principal ideal domain, thus J is principal, i.e., J = ( f ) for some
f ∈ k[x]. In addition, since k is algebraically closed, there exist n ∈ N, c, a1 , . . . , an ∈ k such that

p(x) = c(x − a1 ) · · · (x − an ).

Therefore, we see that


Z(p) = {a1 , . . . , an } = Z(J).
Since J is arbitrary, we conclude that all the closed sets are finite, so the open sets are complements
of finite sets.
Since open sets are complements of finite sets, they intersect in infinitely many points, so the
Zariski topology is not Hausdorff.
Recall that the Hausdorff condition on a topological space X is characterized by the fact that the
diagonal
∆ := {(x, x) : x ∈ X}
is closed in the product space X × X. In our context, the diagonal

∆ = {(x, x) : x ∈ k}

is not closed in the product topology of k2 . Nevertheless, ∆ is closed in the Zariski topology, i.e., ∆ is
an algebraic set. Indeed, if we write
p(x, y) := x − y,
then

∆ = {(x, x) ; x ∈ k}
= {(x, y) ; x, y ∈ k, x = y}
= {(x, x) ; p(x, y) = 0}
= Z((p)).

Thus, ∆ is indeed an algebraic set.

Solution of c. (Part 1) Let U be an arbitrary open set. Let p ∈ U be arbitrary. Since U is open, U c is
an algebraic set, i.e., there exists an ideal I in R such that

U c = Z(I) = {J prime ideal of R ; I ⊆ J}.

/ U c , we have that I ⊈ p. So, we can take an element f ∈ I\p. Therefore, p ∈ U( f ). To finish,


Since p ∈
we shall show that
U( f ) ⊆ U.
If p′ ∈ U( f ), then f ∈
/ p′ . Since f ∈ I, it holds that I ⊈ p′ ; so, p ∈
/ U c . This means that any element in
U( f ) belongs to U.
Since U is arbitrary, we have proven that any open set contains a distinguished open set. So, the
family of distinguished open sets form a basis for the Zariski topology. Hence, any open set is the
union of distinguished open sets.
(Part 2) First, we assume that there exists a family { fi }i of points in R such that
[
Spec R = U( fi ). (10)
i

We need to show that the ideal generated by all the fi is the unit ideal (1) = R. By contradiction, we
suppose that I ̸= R. This guarantees the existence of a maximal ideal p containing I. Since p is prime,
for being maximal, and I ⊆ p, it holds that

fi ∈ p for all i.

Now, by (10), we see that \


0/ = Spec Rc = U( fi )c ,
i
so there does not exist a prime ideal q such that fi ∈ q for every i; but we have p with precesily this
property! Since we have reached a contradiction, it follows that

I = ( fi )i = (1).

To finish this exercise, we suppose that { fi } is a family of points in R such that the ideal generated by
all the elements in this family is the unit ideal (1). We shall prove that
\
0/ = U( fi )c .
i

By contradiction, let p be a prime ideal such that p ∈ U( fi )c for every i. It means that

fi ∈ p for all i,

which implies that


( fi )i = (1) ⊆ p,
which is not possible, because p is prime. So, we have that
\ [
0/ = U( fi )c ⇔ Spec R = U( fi ).
i i

Solution of d. Let (Ui )i be an arbitrary open cover for Spec R. Since the family of distinguished open
sets is a basis for the Zariski topology of Spec R, we can assume, without loss of generality, that
Ui = U( fi ) for some fi ∈ R, for every i. So, we have that
[
Spec R = U( fi ).
i

By Part 2 in item c, we know that { fi } is a family of points in R such that the ideal generated by all
the elements in this family is the unit ideal (1). If particular, there exist ai1 , . . . , ain ∈ R such that

ai1 fi1 + · · · + ain fin = 1.

So { fi1 , . . . , fin } generates (1), and by Part 2 in item c, it is equivalent to say that
n
[
Spec R = U( fi j ).
j=1

This means that {U( fi1 ), . . . ,U( fin )} is a finite subcover of (Ui )i for Spec R. Since (Ui )i is arbitrary,
we have shown that every open cover for Spec R admits a finite subcover, i.e., Spec R is compact.

You might also like